1 Introduction

In this paper, we study the existence of a (positive) solution for the following quasi-linear elliptic equation:

$$ (\mathrm{P})\quad \left \{ \textstyle\begin{array}{@{}l@{\quad}l} -\varDelta _{p} u+{\lambda_{1}}|u|^{p-2}u-\mu \varDelta _{q}u +\mu\lambda_{2}|u|^{q-2}u=f(x,u,\nabla u) &\mbox{in } \mathbb{R}^{N}, \\ u>0&\mbox{in } \mathbb{R}^{N}. \end{array}\displaystyle \right . $$

In the left-hand side of the equation in (P), we have the p-Laplacian \(\varDelta _{p}\) and the q-Laplacian \(\varDelta _{q}\) with \(1< q< p<+\infty\) and the constants \(\mu\geq0\), \(\lambda_{1}>0\), and \(\lambda_{2}> 0\). The problem covers the corresponding statement with p-Laplacian in the principal part, for which it is sufficient to take \(\mu=0\). Here \(-\varDelta _{p}\) is regarded as the operator \(-\varDelta _{p}:W^{1,p}(\mathbb{R}^{N})\to W^{-1,p'}(\mathbb{R}^{N})\), where \(\frac{1}{p}+\frac{1}{p'}=1\), defined by

$$\begin{aligned} \langle-\varDelta _{p}u,v\rangle= \int_{\mathbb{R}^{N}}|\nabla u|^{p-2}\nabla u\nabla v \,dx \quad \mbox{for all }u,v\in W^{1,p}\bigl(\mathbb{R}^{N}\bigr). \end{aligned}$$

The right-hand side of the equation in (P) is in the form of convection term, meaning a nonlinearity that depends on the point x in \(\mathbb{R}^{N}\), on the solution u, and on its gradient ∇u.

The existence of positive solutions for problems with p-Laplacian and convection term on a bounded domain has been studied in [13]. In the case where the principal part of the equation is driven by the \((p, q)\)-Laplacian operator with \(1 < q < p\) and by a nonhomogeneous operator, the existence of a positive solution of elliptic problems with convex term on a bounded domain has been investigated in [4] and [5], respectively. Results of this type when the principal part of the equation is expressed through a general Leray-Lions operator can be found in [6]. Essential features of the present work are the dependence on the gradient ∇u, which prevents the use of variational methods, and the unboundedness of the domain, which produces lack of compactness.

We assume that \(f: \mathbb{R}^{N}\times\mathbb{R}\times\mathbb {R}^{N}\to \mathbb{R}\) is a continuous function satisfying the growth condition:

  1. (F0)

    \(f(x,0,\xi)\equiv0\) for all \(x\in\mathbb{R}^{N}, \xi\in\mathbb{R}^{N}\);

  2. (F1)

    there exist constants \(r_{1},r_{2}\in(0,p-1)\) and continuous nonnegative functions \(a_{0}\in L^{p^{\prime}}(\mathbb{R}^{N})\) and \(a_{i}\in L^{\tilde{r}_{i}}(\mathbb{R}^{N})\) (\(i=1,2\)), where \(1/p+1/p^{\prime}=1\) and \(\tilde{r}_{i}=p/(p-r_{i}-1)=(p/(r_{i}+1))^{\prime}\) (\(i=1,2\)), such that

    $$ \bigl|f(x,t,\xi)\bigr|\leq a_{0}(x)+a_{1}(x)|t|^{r_{1}}+a_{2}(x)| \xi|^{r_{2}} $$
    (1)

    for all \((x,t,\xi)\in\mathbb{R}^{N}\times\mathbb{R}\times\mathbb{R}^{N}\).

In this setting, by a solution of problem (P) we mean any function \(u\in W^{1,p}(\mathbb{R}^{N})\cap W^{1,q}(\mathbb{R}^{N})\) when \(\mu>0\) or \(u\in W^{1,p}(\mathbb{R}^{N})\) when \(\mu=0\) such that \(u(x)>0\) for a.e. \(x\in\mathbb{R}^{N}\) and

$$\begin{aligned} & \int_{\mathbb{R}^{N}} \bigl(|\nabla u|^{p-2}+\mu|\nabla u|^{q-2}\bigr)\nabla u \nabla\varphi \,dx+ \int_{\mathbb{R}^{N}} \bigl(\lambda_{1} |u|^{p-2}+\mu \lambda_{2} |u|^{q-2}\bigr)u\varphi \,dx \\ &\quad= \int_{\mathbb{R}^{N}}f(x,u,\nabla u)\varphi \,dx \quad\mbox{for all } \varphi\in C_{0}^{\infty}\bigl(\mathbb{R}^{N}\bigr). \end{aligned}$$

In order to show the positivity of a solution, we will need an additional growth condition when t is small:

  1. (F2)

    there exist constants \(\delta_{0}>0\) and \(r_{0}\in(0,p-1)\) if \(\mu=0\) or \(r_{0}\in(0,q-1)\) if \(\mu>0\) and a continuous positive function \(b_{0}\) such that

    $$ b_{0}(x)t^{r_{0}}\leq f(x,t,\xi) \quad\mbox{for all } 0< t\le\delta_{0}, x\in\mathbb{R}^{N}, \xi\in \mathbb{R}^{N}. $$
    (2)

We mention that the fact that condition (F2) is supposed only for \(t>0\) small is a significant improvement with respect to all the previous works. A direct consequence is that f is allowed to change sign.

For example, the following nonlinearity satisfies our assumptions \((\mathrm{F}0)\sim(\mathrm{F}2)\):

$$f(x,t,\xi)=a_{1}(x)|t|^{r_{0}} +a_{2}(x)| \xi|^{r_{2}} \sin t , $$

with \(a_{1}\in L^{p^{\prime}}(\mathbb{R}^{N})\cap L^{\tilde{r}_{1}}(\mathbb{R}^{N})\) and \(a_{2}\in L^{\tilde{r}_{2}}(\mathbb{R}^{N})\), where \(\tilde{r}_{i}=p/(p-r_{i}-1)\) (\(i=1,2\)).

Our main result provides the existence of a (positive) solution for problem (P).

Theorem 1

Under assumptions (F0)-(F2), problem (P) admits a (positive) solution \(u\in W^{1,p}(\mathbb{R}^{N})\cap W^{1,q}(\mathbb{R}^{N})\cap C^{1}_{\mathrm{loc}}(\mathbb{R}^{N})\) if \(\mu>0\) and \(u\in W^{1,p}(\mathbb{R}^{N})\cap C^{1}_{\mathrm{loc}}(\mathbb{R}^{N})\) if \(\mu=0\).

The proof is based on a priori estimates obtained through hypotheses (F0)-(F2) for approximate solutions on bounded domains and the use of comparison arguments. In this respect, we establish several comparison principles that ultimately determine the positivity of solutions.

Under a stronger version of the growth condition (F1), we show that any positive solution disappear at infinity.

Theorem 2

Assume (F0)-(F2). If one of the following conditions holds, then any (positive) solution u of problem (P) satisfies that \(u(x)\to0\) as \(|x|\to\infty\):

  1. (i)

    \(N\le p\);

  2. (ii)

    \(N>p\) and \(p^{2}< p^{*}\) (if and only if \(p< N< p^{2}/(p-1)\));

  3. (iii)

    \(N\ge p^{2}/(p-1)\), and for the functions \(a_{i}\) (\(i=0,1,2\)) in (F1), there exist \(R_{*}>0\) and \(\gamma_{i}\) such that

    $$\begin{aligned}& \gamma_{0}, \gamma_{1} >\frac{p^{*}}{p^{*}-p},\qquad \gamma_{2}>\frac{pp^{*}}{(p-r_{2})(p^{*}-p)}, \end{aligned}$$
    (3)
    $$\begin{aligned}& \sup_{x_{0}\in\mathbb{R}^{N}} \int_{B(x_{0},2R_{*})} \bigl|a_{i}(x)\bigr|^{\gamma_{i}}\,dx< \infty \end{aligned}$$
    (4)

    for \(i=0,1,2\), where \(p^{*}:=pN/(N-p)\) if \(N> p\).

Since we are looking for positive solutions of problem (P), without any loss of generality, we will suppose in the sequel that \(f(x,t,s)\equiv0\) for all \(t\leq0\) and \((x,s)\in \mathbb{R}^{N}\times\mathbb{R}^{N}\).

The rest of the paper is organized as follows. In Section 2, we present comparison principles related to problem (P). Section 3 deals with approximate solutions on bounded domains. Section 4 is devoted to the proof of Theorem 1. In Section 5, we give a proof of Theorem 2 after we show the boundedness of a solution.

2 Comparison principles

In this section, we assume that D is a bounded domain in \(\mathbb{R}^{N}\). We consider the operator denoted \(-\varDelta _{p}\) from \(W^{1,p}(D)\) to \(W^{1,p}(D)^{*}\) defined by

$$\begin{aligned} \langle-\varDelta _{p}u,v\rangle= \int_{D}|\nabla u|^{p-2}\nabla u\nabla v \,dx \quad \mbox{for all }u,v\in W^{1,p}(D). \end{aligned}$$

First we recall the following result.

Lemma 1

([7], Lemma 2.1)

Let \(w_{1}, w_{2}\in L^{\infty}(D)\) satisfy \(w_{i}\geq0\) a.e. on D, \(w_{i}^{1/q}\in W^{1,p}(D)\) for \(i=1,2\), and \(w_{1}=w_{2}\) on ∂D. If \(w_{1}/w_{2}, w_{2}/w_{1}\in L^{\infty}(D)\), then

$$\begin{aligned} 0\le \biggl\langle -\varDelta _{p} w_{1}^{1/q}- \mu \varDelta _{q} w_{1}^{1/q}, \frac{w_{1}-w_{2}}{w_{1}^{(q-1)/q}} \biggr\rangle - \biggl\langle -\varDelta _{p} w_{2}^{1/q}-\mu \varDelta _{q} w_{2}^{1/q}, \frac{w_{1}-w_{2}}{w_{2}^{(q-1)/q}} \biggr\rangle . \end{aligned}$$
(5)

Lemma 1 leads to a comparison principle for a subsolution and a supersolution of the problem

$$ \left \{ \textstyle\begin{array}{@{}l@{\quad}l} -\varDelta _{p} u+\lambda_{1} |u|^{p-2}u-\mu \varDelta _{q}u+\mu\lambda_{2} |u|^{q-2}u=g(u)& \mbox{in } D,\\ u=0& \mbox{on } \partial D, \end{array}\displaystyle \right . $$
(6)

where \(g:\mathbb{R}\rightarrow\mathbb{R}\) is a continuous function.

We say that \(u_{1}\in W^{1,p}({D})\) is a subsolution of problem (6) if \(u_{1}\leq0\) on ∂D and

$$\begin{aligned}[b] &\int_{D}(|\nabla u_{1}|^{p-2}\nabla u_{1}\nabla\varphi+ \lambda_{1} |u_{1}|^{p-2}u_{1} \varphi+\mu|\nabla u_{1}|^{q-2}\nabla u_{1}\nabla \varphi+ \mu\lambda_{2} |u_{1}|^{q-2}u_{1} \varphi \,dx \\ &\quad \leq \int_{D} g(u_{1})\varphi \,dx \end{aligned} $$

for all \(\varphi\in W^{1,p}_{0}({D})\) with \(\varphi\geq0\) in D, provided that the integral \(\int_{D} g(u_{1})\varphi \,dx\) exists. We say that \(u_{2}\in W^{1,p}({D})\) is a supersolution of (6) if the reversed inequalities are satisfied with \(u_{2}\) in place of \(u_{1}\) for all \(\varphi\in W^{1,p}_{0}({D})\) with \(\varphi\geq0\) in D.

Theorem 3

Let \(g:\mathbb{R}\rightarrow\mathbb{R}\) be a continuous function such that \(t^{1-q}g(t)\) is nonincreasing for \(t>0\) if \(\mu>0\) and \(t^{1-p}g(t)\) is nonincreasing for \(t>0\) if \(\mu=0\). Assume that \(u_{1}\) and \(u_{2}\) are a positive subsolution and a positive supersolution of problem (6), respectively. If \(u_{2}(x)>u_{1}(x)=0\) for all \(x\in\partial D\) and \(u_{i}\in C^{1}(\overline{D})\) for \(i=1,2\), then \(u_{2}\geq u_{1}\) in D.

Proof

We prove the result only for \(\mu>0\) because the case \(\mu=0\) is easier. Suppose by contradiction that the set \({D}_{0}=\{x\in{D}: u_{1}(x)>u_{2}(x)\}\) is nonempty. Let U be a connected component of \({D}_{0}\). Noting that \(\inf_{\overline{D}}u_{2}>0\), we see that \(\overline{U}\subset D\), \(u_{1}=u_{2}\) on ∂U, and \(u_{i}/u_{j}\in L^{\infty}(U)\) for \(i,j=1,2\). So, \((u_{1}^{q}-u_{2}^{q})/u_{1}^{q-1}, (u_{1}^{q}-u_{2}^{q})/u_{2}^{q-1}\in W_{0}^{1,p}(U)\), and extending by 0 on \(D\setminus U\), we can take them as test functions in the above definitions of subsolution and supersolution for problem (6). It follows that

$$\begin{aligned} & \biggl\langle -\varDelta _{p} u_{1}-\mu \varDelta _{q} u_{1}, \frac{u_{1}^{q}-u_{2}^{q}}{u_{1}^{q-1}} \biggr\rangle - \biggl\langle - \varDelta _{p} u_{2}-\mu \varDelta _{q} u_{2}, \frac{u_{1}^{q}-u_{2}^{q}}{u_{2}^{q-1}} \biggr\rangle \\ &\quad\le \int_{U} \bigl(-\lambda_{1} u_{1}^{p-1}- \mu\lambda_{2} u_{1}^{q-1}+g(u_{1})\bigr) \frac{u_{1}^{q}-u_{2}^{q}}{u_{1}^{q-1}}\,dx \\ &\qquad{}- \int_{U} \bigl(-\lambda_{1} u_{2}^{p-1}- \mu\lambda_{2} u_{2}^{q-1}+g(u_{2})\bigr) \frac{u_{1}^{q}-u_{2}^{q}}{u_{2}^{q-1}}\,dx \\ &\quad=-\lambda_{1} \int_{U} \bigl(u_{1}^{p-q}-u_{2}^{p-q} \bigr) \bigl(u_{1}^{q}-u_{2}^{q}\bigr)\,dx + \int_{U} \biggl(\frac{g(u_{1})}{u_{1}^{q-1}}-\frac{g(u_{2})}{u_{2}^{q-1}} \biggr) \bigl(u_{1}^{q}-u_{2}^{q}\bigr) \\ &\quad< 0. \end{aligned}$$

The last inequality is obtained through our assumption that \(g(t)/t^{q-1}\) is nonincreasing for \(t>0\) and \(\lambda_{1}>0\).

On the other hand, note that we can apply Lemma 1 with \(w_{i}=u_{i}^{q}\) (\(i=1,2\)) and U in place of D. The conclusion provided by (5) in Lemma 1 contradicts the above inequality in the case where U is nonempty. Therefore, \({D}_{0}=\emptyset\), which completes the proof. □

The next theorem points out that the condition of supersolution in Theorem 3 can be relaxed to a weaker notion of supersolution directly related to the given subsolution.

Theorem 4

Let \(g:\mathbb{R}\rightarrow\mathbb{R}\) be continuous function such that \(t^{1-q}g(t)\) is nonincreasing for \(t>0\) if \(\mu>0\) and \(t^{1-p}g(t)\) is nonincreasing for \(t>0\) if \(\mu=0\). Assume that \(u_{1}\in C^{1}(\overline{D})\) is a positive subsolution of problem (6) and that \(h:\overline{D}\times\mathbb{R}\times \mathbb{R}^{N}\rightarrow\mathbb{R}\) is continuous function such that \(h(x,t,\xi)\ge g(t)\) for all \(x\in\mathbb{R}^{N}\), \(\xi\in\mathbb{R}^{N}\), and \(t\in(0,\|u_{1}\|_{L^{\infty}(D)} ]\). If \(u_{2}\in C^{1}(\overline{D})\) is a (positive) solution of

$$ \left \{ \textstyle\begin{array}{@{}l@{\quad}l} -\varDelta _{p} u+\lambda_{1} |u|^{p-2}u-\mu \varDelta _{q}u+\mu\lambda_{2} |u|^{q-2}u=h(x,u,\nabla u)& \textit{in } D,\\ u>0& \textit{in } D \end{array}\displaystyle \right . $$
(7)

such that \(u_{2}(x)>u_{1}(x)=0\) for all \(x\in\partial D\), then \(u_{2}\geq u_{1}\) in D.

Proof

The conclusion can be achieved following the same argument as in Theorem 3. Indeed, arguing by contradiction, let us assume that the set \(D_{0}:=\{x\in D : u_{1}(x)>u_{2}(x)\}\) is not empty. Let U be a connected component of \(D_{0}\). Then \(u_{1}=u_{2}\) on ∂U and \(g(u_{2})\le h(x,u_{2},\nabla u_{2})\) in U. Proceeding as in the proof of Theorem 3, we have

$$\begin{aligned} \biggl\langle -\varDelta _{p} u_{1}-\mu \varDelta _{q} u_{1}, \frac{u_{1}^{q}-u_{2}^{q}}{u_{1}^{q-1}} \biggr\rangle - \biggl\langle - \varDelta _{p} u_{2}-\mu \varDelta _{q} u_{2}, \frac{u_{1}^{q}-u_{2}^{q}}{u_{2}^{q-1}} \biggr\rangle < 0. \end{aligned}$$

This leads to a contradiction by applying Lemma 1. □

In the case where \(u_{1}\) and \(u_{2}\) satisfy the homogeneous Dirichlet boundary condition we can state the following:

Theorem 5

Let \(g:\mathbb{R}\rightarrow\mathbb{R}\) be a continuous function such that \(t^{1-q}g(t)\) is nonincreasing for \(t>0\) if \(\mu>0\) and \(t^{1-p}g(t)\) is nonincreasing for \(t>0\) if \(\mu=0\). Assume that \(u_{1}\in C_{0}^{1}(\overline{D})\) is a positive subsolution of problem (6) and that \(h:\overline{D}\times\mathbb{R}\times \mathbb{R}^{N}\rightarrow\mathbb{R}\) is continuous and such that \(h(x,t,\xi)\ge g(t)\) for all \(x\in\mathbb{R}^{N}\), \(\xi\in\mathbb{R}^{N}\), and \(t\in(0, \|u_{1}\|_{L^{\infty}(D)} ]\). If \(u_{2}\in C^{1}_{0}(\overline{D})\) is a (positive) solution of problem (7) such that \(u_{1}/u_{2}\in L^{\infty}(D)\) and \(u_{2}/u_{1}\in L^{\infty}(D)\), then \(u_{2}\geq u_{1}\) in D.

Proof

Due to the assumptions \(u_{1}/u_{2}\in L^{\infty}(D)\) and \(u_{2}/u_{1}\in L^{\infty}(D)\), it turns out that \((u_{1}^{q}-u_{2}^{q})/u_{1}^{q-1}, (u_{1}^{q}-u_{2}^{q})/u_{2}^{q-1}\in W_{0}^{1,p}(U)\) with U introduced in the proof of Theorem 3. Then we can conclude as in the proof of Theorem 4. □

3 Solution on a bounded domain

In this section, we assume that D is a bounded domain in \(\mathbb{R}^{N}\) with \(C^{2}\) boundary ∂D. For \(r\geq1\), we denote by \(\|u\|_{L^{r}(D)}\) the usual norm on the space \(L^{r}(D)\). We endow \(W_{0}^{1,p}(D)\) with the norm \(\|u\|_{D}^{p}=\|\nabla u\|_{L^{p}(D)}^{p}+\lambda_{1}\|u\|_{L^{p}(D)}^{p}\), which is equivalent to the usual one.

We focus on the existence of a (positive) solution for the problem

$$ (\mathrm{PD})\quad \left \{ \textstyle\begin{array}{@{}l@{\quad}l} -\varDelta _{p} u+\lambda_{1} |u|^{p-2}u-\mu \varDelta _{q}u+\mu\lambda_{2} |u|^{q-2}u =f(x,u,\nabla u) &\mbox{in } D,\\ u>0&\mbox{in } D,\\ u(x)= 0 &\mbox{on }\partial D. \end{array}\displaystyle \right . $$

Here we impose the following hypotheses: \(f: \overline{D}\times \mathbb{R}\times\mathbb{R}^{N}\to\mathbb{R}\) is a continuous function satisfying

\((\widetilde{\mathrm{F}0})\) :

\(f(x,0,\xi)\equiv0\) for all \(x\in D, \xi\in\mathbb{R}^{N}\);

\((\widetilde{\mathrm{F}1})\) :

there exist constants \(r_{1},r_{2}\in(0,p-1)\) and continuous nonnegative functions \(a_{i}\) (\(i=0, 1,2\)) on such that

$$ \bigl|f(x,t,\xi)\bigr|\leq a_{0}(x)+a_{1}(x)|t|^{r_{1}}+a_{2}(x)| \xi|^{r_{2}} $$
(8)

for all \((x,t,\xi)\in D\times\mathbb{R}\times\mathbb{R}^{N}\);

\((\widetilde{\mathrm{F}2})\) :

there exist constants \(\delta_{0}>0\) and \(r_{0}\in(0,p-1)\) if \(\mu=0\) or \(r_{0}\in(0,q-1)\) if \(\mu>0\) and a continuous function \(b_{0}\) such that \(\inf_{x\in D} b_{0}(x)>0\) and

$$ b_{0}(x)t^{r_{0}}\leq f(x,t,\xi) \quad\mbox{for all } 0< t\le\delta_{0}, x\in D, \xi\in\mathbb{R}^{N}. $$
(9)

We say that \(u\in W_{0}^{1,p}(D)\) is a solution of (PD) if \(u(x)>0\) for a.e. \(x\in D\) and

$$\begin{aligned} & \int_{D} \bigl(|\nabla u|^{p-2}+\mu|\nabla u|^{q-2}\bigr)\nabla u\nabla\varphi \,dx + \int_{D} \bigl(\lambda_{1} |u|^{p-2}+\mu \lambda_{2}|u|^{q-2}\bigr)u\varphi \,dx \\ &\quad= \int_{D} f(x,u,\nabla u)\varphi \,dx\quad \mbox{for all } \varphi \in W_{0}^{1,p}(D). \end{aligned}$$

The existence of a solution for problem (PD) is stated as follows.

Theorem 6

Under assumptions \((\widetilde{\mathrm{F}0})\)-\((\widetilde{\mathrm{F}2})\), problem (PD) admits a (positive) solution \(u\in C^{1}_{0}(\overline{D})\) such that \(\partial u/\partial\nu<0\) on ∂D, where ν stands for the outer normal to ∂D.

In the proof of Theorem 6, we utilize the following approximate equation:

$$ (\mathrm{PD}_{\varepsilon})\quad \left \{ \textstyle\begin{array}{@{}l@{\quad}l} -\varDelta _{p} u+\lambda_{1} |u|^{p-2}u-\mu \varDelta _{q}u+\mu\lambda_{2}|u|^{q-2}u =f(x,u,\nabla u)+\varepsilon\psi&\mbox{in } D,\\ u>0&\mbox{in } D,\\ u(x)= 0 &\mbox{on }\partial D, \end{array}\displaystyle \right . $$

with \(\varepsilon>0\) and a nonnegative function \(0\not\equiv \psi\in C(\overline{D})\).

Lemma 2

Under \((\widetilde{\mathrm{F}0})\)-\((\widetilde{\mathrm{F}2})\), for any \(\varepsilon>0\) and a nonnegative function \(0\not\equiv\psi\in C(D)\), problem (\(\mathrm{PD}_{\varepsilon}\)) admits a (positive) solution \(u_{\varepsilon}\in C^{1}_{0}(\overline{D})\) such that \(\partial u_{\varepsilon}/\partial\nu<0\) on ∂D.

Proof

We argue as in [5], Proposition 8, and [7]. Fix \(\varepsilon>0\) and consider a Schauder basis \(\{e_{1},\ldots,e_{m},\ldots\}\) of \(W^{1,p}_{0}(D)\) (refer to [4, 8] for its existence). For each \(m\in\mathbb{N}\), we define the m-dimensional subspace \(V_{m}:=\operatorname{span}\{e_{1},\ldots, e_{m}\}\) of \(W^{1,p}_{0}(D)\). The map \(T_{m}\colon\mathbb{R}^{m} \to V_{m}\) defined by \(T_{m}(\xi_{1},\ldots,\xi_{m})=\sum_{i=1}^{m}\xi_{i}e_{i}\) is a linear isomorphism. Let \(T_{m}^{*}:V_{m}^{*}\to(\mathbb{R}^{m})^{*}\) be the dual map of \(T_{m}\). Identifying \(\mathbb{R}^{m}\) and \((\mathbb{R}^{m})^{*}\), we may regard \(T_{m}^{*}\) as a map from \(V_{m}^{*}\) to \(\mathbb{R}^{m}\). Define the maps \(A_{m}\) and \(B_{m}\) from \(V_{m}\) to \(V_{m}^{*}\) as follows:

$$\begin{aligned} \bigl\langle A_{m}(u), v \bigr\rangle := \int_{D} \bigl(|\nabla u|^{p-2}+\mu|\nabla u|^{q-2} \bigr) \nabla u\nabla v \,dx \end{aligned}$$

and

$$\begin{aligned} \bigl\langle B_{m}(u), v \bigr\rangle :=- \int_{D} \bigl(\lambda_{1}|u|^{p-2}+\mu \lambda_{2} |u|^{q-2} \bigr)uv \,dx + \int_{D} \bigl(f(x,u,\nabla u)+\varepsilon\psi \bigr) v \,dx \end{aligned}$$

for all u, \(v\in V_{m}\).

By \((\widetilde{\mathrm{F}1})\) and Hölder’s inequality, we have

$$\begin{aligned} &\bigl\langle A_{m}(u)-B_{m}(u), u \bigr\rangle \\ &\quad\ge \|u\|_{D}^{p}-d\bigl(\|u\|_{L^{1}(D)}+\|u \|_{L^{r_{1}+1}(D)}^{r_{1}+1} +\|u\|_{D}^{r_{2}+1}\bigr)- \varepsilon\|\psi\|_{L^{\infty}(D)}\|u\| _{L^{1}(D)} \end{aligned}$$
(10)

for all \(u\in V_{m}\), where d is a positive constant independent of m and u. Because of \(r_{1}+1< p\) and \(r_{2}+1< p\), we easily see that \(A_{m}-B_{m}\) is coercive on \(V_{m}\), whence \(T_{m}^{*}\circ(A_{m}-B_{m})\circ T_{m}\) is coercive on \(\mathbb{R}^{m}\). By a well-known consequence of Brouwer’s fixed point theorem it follows that there exists \(y_{m}\in\mathbb{R}^{m}\) such that \((T_{m}^{*}\circ(A_{m}-B_{m})\circ T_{m} )(y_{m})=0\), and hence \(A_{m}(u_{m})-B_{m}(u_{m})=0\) with \(u_{m}=T_{m}(y_{m})\in V_{m}\).

Writing (10) with \(u=u_{m}\in W^{1,p}_{0}(D)\) shows the boundedness of the sequence \(\|u_{m}\|_{D}\). Thus, along a subsequence, \(u_{m}\) converges to some \(u_{0}\) weakly in \(W^{1,p}_{0}(D)\) and strongly in \(L^{p}(D)\).

We claim that

$$ u_{m}\to u_{0} \quad\mbox{in } W_{0}^{1,p}(D) \mbox{ as } m\to\infty. $$
(11)

Let \(P_{m}\) denote the projection onto \(V_{m}\), that is, \(P_{m} u=\sum_{i=1}^{m}\xi_{i}e_{i}\) for \(u=\sum_{i=1}^{\infty}\xi_{i}e_{i}\). Since \(u_{m}\), \(P_{m}u_{0}\in V_{m}\) and \(A_{m}(u_{m})-B_{m}(u_{m})=0\) in \(V_{m}^{*}\), we obtain

$$\begin{aligned} &\bigl\langle A_{m}(u_{m}), u_{m}-P_{m}u_{0} \bigr\rangle \\ &\quad=\bigl\langle B_{m}(u_{m}), u_{m}-P_{m}u_{0} \bigr\rangle \\ &\quad=\bigl\langle B_{m}(u_{m}), u_{m}-u_{0} \bigr\rangle +\bigl\langle B_{m}(u_{m}), u_{0}-P_{m}u_{0} \bigr\rangle \\ &\quad= \int_{D} \bigl(-\lambda_{1}|u_{m}|^{p-2}u_{m}- \mu\lambda_{2} |u_{m}|^{q-2}u_{m}+f(x,u_{m}, \nabla u_{m}) \bigr) (u_{m}-P_{m}u_{0}) \,dx \\ &\qquad{}+ \int_{D}\varepsilon\psi(u_{m}-P_{m}u_{0}) \,dx \\ &\quad\to0\quad \mbox{as } m\to\infty, \end{aligned}$$

where we use \((\widetilde{\mathrm{F}1})\), the boundedness of \(\|u_{m}\|_{D}\), \(u_{m}\to u_{0}\) in \(L^{p}(\Omega)\), and \(P_{m}u_{0}\to u_{0}\) in \(W^{1,p}_{0}(D)\). This leads to

$$ \lim_{m\to\infty} \int_{D}|\nabla u_{m}|^{p-2}\nabla u_{m}\nabla (u_{m}-u_{0})\,dx=0. $$

In view of the \((S_{+})\)-property of \(-\varDelta _{p}\) (see, e.g., [9], Proposition 3.5, or refer to (22) in the proof of Theorem 1), we obtain (11).

Now let us prove that \(u_{0}\) is a solution of (\(\mathrm{PD}_{\varepsilon}\)). Fix \(l\in\mathbb{N}\) and \(\varphi\in V_{l}\). For each \(m\ge l\), letting \(m\to\infty\) in \(\langle A_{m}(u_{m}),\varphi\rangle=\langle B_{m}(u_{m}),\varphi\rangle\) and making use of (11), we have

$$\begin{aligned} & \int_{D} \bigl(|\nabla u_{0}|^{p-2}+\mu| \nabla u_{0}|^{q-2}\bigr)\nabla u_{0} \nabla \varphi \,dx+ \int_{D} \bigl(\lambda_{1} |u_{0}|^{p-2}+ \mu\lambda_{2} |u_{0}|^{q-2}\bigr)u_{0} \varphi \,dx \\ &\quad= \int_{D} \bigl(f(x,u_{0},\nabla u_{0})+\varepsilon\psi \bigr) \varphi \,dx. \end{aligned}$$
(12)

Since l is arbitrary, equality (12) holds for every \(\varphi\in\bigcup_{l\ge1} V_{l}\). In fact, the density of \(\bigcup_{l\ge 1} V_{l}\) in \(W^{1,p}_{0}(D)\) guarantees that (12) holds for every \(\varphi\in W^{1,p}_{0}(D)\). This means that \(u_{0}\) is a solution of (\(\mathrm{PD}_{\varepsilon}\)). Acting with \(-u_{0}^{-}\) (where \(u_{0}^{-}:=\max\{0,-u_{0}\}\)) and taking into account that \(\psi\ge0\) and \(f(x,t,\xi)=0\) for \(t\le0\), we see that

$$\begin{aligned} &\bigl\| u_{0}^{-}\bigr\| _{D}^{p}+\mu\bigl\| \nabla u_{0}^{-}\bigr\| _{L^{q}(D)}^{q} +\mu\lambda_{2} \bigl\| u_{0}^{-}\bigr\| _{L^{q}(D)}^{q} \\ &\quad= \int_{u_{0}< 0} \bigl(f(x,u_{0},\nabla u_{0})+\varepsilon\psi \bigr) u_{0} \,dx =\varepsilon \int_{u_{0}< 0}\psi u_{0} \,dx \le 0, \end{aligned}$$

whence \(u_{0}\ge0\) a.e. in D. Moreover, \(u_{0}\not\equiv0\) because we assumed that \(\psi\not\equiv0\) and \(\varepsilon>0\). Next, we observe that hypothesis \((\widetilde{\mathrm{F}1})\) allows us to refer to [10], Theorem 7.1 (see also [11] and [5]), from which we infer that \(u\in L^{\infty}(D)\). Furthermore, the regularity result up to the boundary in [12], Theorem 1, and [13], p.320, ensures that \(u\in C_{0}^{1,\beta}(\overline{D})\) with some \(\beta\in(0,1)\). Applying the strong maximum principle in [14], Theorem 5.4.1, and the boundary point lemma in [14], Theorem 5.5.1 (note that \(f(x,t,\xi)\ge0\) for \(0\le t\le \delta_{0}\)) entails that \(u>0\) in D and \(\partial u/\partial\nu<0\) on ∂D. Altogether, we have established that the conclusion of lemma is fulfilled for \(u_{\varepsilon}=u_{0}\). □

We will also need the following result.

Lemma 3

Let \(1< q< p<+\infty\), \(\lambda_{1}>0\), \(\lambda_{2}\ge0\), and \(\mu\geq0\). For any constants \(b>0\) and \(0< r< p-1\) with \(0< r< q-1\) if \(\mu>0\), the problem

$$ \left \{ \textstyle\begin{array}{@{}l@{\quad}l} -\varDelta _{p} u+\lambda_{1}|u|^{p-2}u-\mu \varDelta _{q}u+\mu\lambda_{2} |u|^{q-2}u=bu^{r} &\textit{in }D,\\ u>0&\textit{in } D,\\ u=0 & \textit{on } \partial D \end{array}\displaystyle \right . $$
(13)

admits a solution \(u_{b}\in C^{1}_{0}(\overline{D})\) satisfying \(\lambda_{1}\|u_{b}\|_{L^{\infty}(D)}^{p-r-1}\le b\) and \(\partial u_{b}/\partial\nu<0\) on ∂D.

Proof

We can proceed for the existence of a solution of (13) along the lines of the proof of [7], Lemma 3. For readers’ convenience, we outline the proof in the case where \(\mu>0\). Given the constants \(b>0\) and \(0< r< q-1\), we define the functional \(I:W^{1,p}_{0}(D)\to\mathbb{R}\) by

$$\begin{aligned} I(u) =\frac{1}{p} \int_{D}\bigl(|\nabla u|^{p}+\lambda_{1} |u|^{p}\bigr)\,dx +\frac{\mu}{q} \int_{D}\bigl(|\nabla u|^{q}+\lambda_{2} |u|^{q}\bigr)\,dx -\frac{b}{r+1} \int_{D}\bigl(u^{+}\bigr)^{r+1}\,dx \end{aligned}$$

for all \(u\in W^{1,p}_{0}(D)\), where \(u^{+}=\max\{0,u\}\). Notice that I is of class \(C^{1}\). By using the Sobolev embedding theorem we have the estimate

$$I(u)\geq\frac{1}{p}\|u\|^{p}_{D}-c\|u \|^{r+1}_{D} \quad \mbox{for all }u\in W^{1,p}_{0}(D) $$

with a constant \(c>0\) independent of u. Since \(p>r+1\), I is bounded from below and coercive. Having that I is sequentially weakly lower semicontinuous too, there exists \(u_{b}\in W^{1,p}_{0}(D)\) such that

$$I(u_{b})=\inf_{u\in W^{1,p}_{0}(D)}I(u) $$

(see, e.g., [15], Theorems 1.1, 1.2). In addition, because of \(r+1< q< p\), taking any positive smooth function v and a sufficiently small \(t>0\), we have

$$\begin{aligned} I(tv)&=t^{r+1} \biggl( \frac {t^{p-r-1}}{p}\|v\|_{D} + \frac{\mu t^{q-r-1}}{q} \bigl(\|\nabla v\|_{L^{q}(D)}^{q} + \lambda_{2}\|v\|_{L^{q}(D)}^{q} \bigr) - \frac{\|v\|_{L^{r+1}(D)}^{r+1}}{r+1} \biggr) < 0. \end{aligned}$$

This ensures that \(\inf_{u\in W^{1,p}_{0}(D)}I(u) <0\), and hence \(u_{b}\) is a nontrivial critical point of I. By the regularity theory we infer that \(u_{b}\in C^{1}_{0}(\overline{D})\). Taking \(-u_{b}^{-}\) as a test function in the equation \(I'(u_{b})=0\), we see that \(u_{b}\ge0\). Then the strong maximum principle enables us to derive that \(u_{b}>0\) in D, so \(u_{b}\) is a solution of problem (13), and \(\partial u_{b}/\partial\nu<0\) on ∂D.

Taking \(u_{b}^{\alpha+1}\) with \(\alpha>0\) as a test function in (13), by using Hölder’s inequality and that \(r+1< p\) we get

$$\lambda_{1}\|u_{b}\|_{L^{p+\alpha}(D)}^{p+\alpha} \le b \int_{D} u_{b}^{r+\alpha+1}\,dx \le b \|u_{b}\|_{L^{p+\alpha}(D)}^{r+\alpha+1}|D|^{(p-r-1)/(p+\alpha)}, $$

where \(|D|\) denotes the Lebesgue measure of D, and hence

$$\lambda_{1}\|u_{b}\|_{L^{p+\alpha}(D)}^{p-r-1} \le b|D|^{(p-r-1)/(p+\alpha)}. $$

Letting \(\alpha\to\infty\), we conclude that \(\lambda_{1} \|u_{b}\|_{L^{\infty}(D)}^{p-r-1} \le b\). □

Proof of Theorem 6

Using the data \(\delta_{0}\), \(r_{0}\), and \(b_{0}\) in \((\widetilde{\mathrm{F}2})\), we fix a positive constant b such that

$$ b\le\min \Bigl\{ \inf_{x\in D}b_{0}(x), \lambda_{1}\delta^{p-r_{0}-1} \Bigr\} . $$
(14)

Then, according to Lemma 3, there exists a (positive) solution \(u_{b}\) of

$$ \left \{ \textstyle\begin{array}{@{}l@{\quad}l} -\varDelta _{p} u+\lambda_{1}|u|^{p-2}u-\mu \varDelta _{q}u+\mu\lambda_{2} |u|^{q-2}u=bu^{r_{0}} &\mbox{in }D,\\ u>0&\mbox{in } D,\\ u=0 & \mbox{on } \partial D, \end{array}\displaystyle \right . $$

satisfying

$$ \|u_{b}\|_{L^{\infty}(D)}\le \biggl(\frac{b}{\lambda_{1}} \biggr)^{\frac {1}{p-r_{0}-1}}\le \delta_{0} $$
(15)

(note (14)). Let \(u_{\varepsilon}\) (\(\varepsilon>0\)) be a positive solution of problem (\(\mathrm{PD}_{\varepsilon}\)) obtained by Lemma 2. Let us observe that \(u_{b}/u_{\varepsilon}, u_{\varepsilon}/u_{b} \in L^{\infty}(D)\) because \(u_{b}\) and \(u_{\varepsilon}\) are positive functions belonging to \(C^{1}_{0}(\overline{D})\) and satisfying \(\partial u_{i}/\partial\nu<0\) on ∂D (\(i=b\) and \(i=\varepsilon\)). On the basis of (15), we are able to apply Theorem 5 with \(u_{1}=u_{b}\), \(u_{2}=u_{\varepsilon}\), \(g(t)=b t^{r_{0}}\), and \(h(x,t,\xi)=f(x,t,\xi)+\varepsilon\psi\) because for any \(0< t\le\|u_{b}\|_{L^{\infty}(D)}\), we have that

$$ h(x,t,\xi)=f(x,t,\xi)+\varepsilon\psi\ge f(x,t,\xi) \ge bt^{r_{0}}=g(t) $$

by (14) and \((\widetilde{\mathrm{F}2})\). In this way, we see that \(u_{\varepsilon}\ge u_{b}\) in D for every \(\varepsilon>0\).

Using the growth condition (8) of f, taking \(u_{\varepsilon}\) as a test function in (\(\mathrm{PD}_{\varepsilon}\)), we obtain the inequality

$$\begin{aligned} \|u_{\varepsilon}\|_{D}^{p} \le& \int_{D} \bigl(a_{0} u_{\varepsilon}+a_{1} u_{\varepsilon}^{r_{1}+1} +a_{2} |\nabla u_{\varepsilon}|^{r_{2}}u_{\varepsilon}\bigr)\,dx +\varepsilon\|\psi\|_{L^{p^{\prime}}(D)}\|u_{\varepsilon}\|_{L^{p}(D)} \\ \le& \|a_{0}\|_{L^{p^{\prime}}(D)}\|u_{\varepsilon}\|_{L^{p}(D)} +\|a_{1}\|_{L^{\tilde{r}_{1}}(D)}\|u_{\varepsilon}\|_{L^{p}(D)}^{r_{1}+1} \\ &{}+\|a_{2}\|_{L^{\tilde{r}_{2}}(D)}\|\nabla u_{\varepsilon}\|_{L^{p}(D)}^{r_{2}} \|u_{\varepsilon}\|_{L^{p}(D)} + \varepsilon\|\psi\|_{L^{p^{\prime}}(D)}\|u_{\varepsilon}\|_{L^{p}(D)} \\ \le& \lambda_{1}^{-1/p}\|a_{0}\|_{L^{p^{\prime}}(D)} \|u_{\varepsilon}\|_{D} +\lambda_{1}^{-(r_{1}+1)/p} \|a_{1}\|_{L^{\tilde{r}_{1}}(D)}\|u_{\varepsilon}\|_{D}^{r_{1}+1} \\ &{}+\lambda_{1}^{-1/p} \|a_{2}\|_{L^{\tilde{r}_{2}}(D)} \|u_{\varepsilon}\|_{D}^{r_{2}+1} +\lambda_{1}^{-1/p} \varepsilon\|\psi\|_{L^{p^{\prime}}(D)}\| u_{\varepsilon}\|_{D} \end{aligned}$$

for every \(\varepsilon>0\). This shows the boundedness of \(\{u_{\varepsilon}\}_{\varepsilon\in(0,1]}\) in \(W_{0}^{1,p}(D)\) because \(p>r_{1}+1, r_{2}+1\) (note \(0<\varepsilon\le1\)). Thus, we can find a sequence \(\varepsilon_{n}\to0^{+}\) such that \(u_{n}:=u_{\varepsilon_{n}}\) is weakly convergent to some u in \(W^{1,p}_{0}(D)\) and strongly in \(L^{r}(D)\) (for all \(r\in[1,p^{*})\)). On the other hand, taking \(u_{n}-u\) as a test function, we easily see that

$$\begin{aligned} U_{n} := & \int_{D} \bigl(|\nabla u_{n}|^{p-2}+\mu| \nabla u_{n}|^{q-2}\bigr) \nabla u_{n} \nabla(u_{n}-u)\,dx \\ \le& -\lambda_{1} \int_{D} u_{n}^{p-1}(u_{n}-u) \,dx -\mu\lambda_{2} \int_{D} u_{n}^{q-1}(u_{n}-u) \,dx \\ &{} + \int_{D} \bigl(a_{0}+a_{1} u_{n}^{r_{1}}+a_{2} |\nabla u_{n}|^{r_{2}} \bigr) (u_{n}-u)\,dx \\ &{}+\varepsilon_{n} \|\psi\|_{L^{p^{\prime}}(D)} \|u_{n}-u \|_{L^{p}(D)} \\ \le& \lambda_{1}\|u_{n}\|_{L^{p}(D)}^{p-1} \|u_{n}-u\|_{L^{p}(D)} +\mu\lambda_{2} \|u_{n}\|_{L^{q}(D)}^{q-1}\|u_{n}-u \|_{L^{q}(D)} \\ &{}+\|a_{0}\|_{L^{p^{\prime}}(D)}\|u_{n}-u\|_{L^{p}(D)} +\|a_{1}\|_{L^{\infty}(D)}\|u_{n}\|_{L^{p^{\prime}r_{1}}(D)}^{r_{1}} \|u_{n}-u\|_{L^{p}(D)} \\ &{} +\|a_{2}\|_{L^{\tilde{r}_{2}}(D)}\|\nabla u_{n} \|_{L^{p}(D)}^{r_{2}} \|u_{n}-u\|_{L^{p}(D)} + \varepsilon\|\psi\|_{L^{p^{\prime}}(D)}\|u_{n}-u\|_{L^{p}(D)} \\ \to&0\quad \mbox{as } n\to\infty. \end{aligned}$$

Hence, \(\limsup_{n\to\infty} U_{n}\le0\). According to the \((S_{+})\) property of \(-\varDelta _{p}\), this ensures that \(u_{n}\) is strongly convergent to u in \(W^{1,p}_{0}(D)\) (refer to (22)). Hence, u is a solution of (PD). Since we already know that \(u_{n}\ge u_{b}\) in D for every n, in the limit, we obtain that \(u\ge u_{b}\) in D. This completes the proof. □

4 Proof of Theorem 1

In this section, we denote \(B_{n}:=B_{n}(0)\) the open ball with center at the origin and radius n. The spaces \(W^{1,p}(B_{n})\) and \(W^{1,q}(B_{n})\) are equipped with the norms

$$\|u\|_{p,n}^{p} := \int_{B_{n}} \bigl(|\nabla u|^{p}+\lambda _{1}|u|^{p} \bigr)\,dx $$

and

$$\|u\|_{q,n}^{q}:= \int_{B_{n}} \bigl(|\nabla u|^{q}+ \lambda_{2} |u|^{q} \bigr)\,dx, $$

respectively.

Proof of Theorem 1

By applying Theorem 6 with \(D=B_{n}\) (\(n\in\mathbb{N}\)) we obtain a (positive) solution \(v_{n}\in C_{0}^{1}(\overline{B_{n}})\) of the problem

$$ (\mathrm{P}_{n})\quad \left \{ \textstyle\begin{array}{@{}l@{\quad}l} -\varDelta _{p} u+\lambda_{1} |u|^{p-2}u-\mu \varDelta _{q}u+\mu\lambda_{2} |u|^{q-2}u =f(x,u,\nabla u) &\mbox{in } B_{n},\\ u>0&\mbox{in } B_{n},\\ u(x)= 0 &\mbox{on }\partial B_{n}. \end{array}\displaystyle \right . $$

We claim that there exists a positive constant C such that

$$ \|v_{n}\|_{p,n} \le C \quad\mbox{for all } n\in\mathbb{N} $$
(16)

and

$$ \|v_{n}\|_{q,n}\le C \quad\mbox{for all } n\in\mathbb{N}, \mbox{provided that } \mu>0. $$
(17)

Indeed, acting with \(v_{n}\) in (\(\mathrm{P}_{n}\)) as a test function, through assumption (F1) and Hölder’s and Young’s inequalities we obtain

$$\begin{aligned} \|v_{n}\|_{p,n}^{p}+\mu\|v_{n} \|_{q,n}^{q} =& \int_{B_{n}} f(x,v_{n},\nabla v_{n})v_{n} \,dx \\ \le& \|a_{0}\|_{L^{p^{\prime}}(B_{n})}\|v_{n}\|_{L^{p}(B_{n})} +\|a_{1}\|_{L^{\tilde{r}_{1}}(B_{n})}\|v_{n}\|_{L^{p}(B_{n})}^{r_{1}+1} \\ &{} +\|a_{2}\|_{L^{\tilde{r}_{2}}(B_{n})}\|v_{n} \|_{L^{p}(B_{n})} \|\nabla v_{n}\|_{L^{p}(B_{n})}^{r_{2}} \\ \leq& \frac{\lambda_{1}}{2}\|v_{n}\|_{L^{p}(B_{n})}^{p}+ \frac{1}{2} \|\nabla v_{n}\|_{L^{p}(B_{n})}^{p} \\ &{} +C \bigl(\|a_{0}\|_{L^{p^{\prime}}(B_{n})}^{p^{\prime}} +\|a_{1}\|_{L^{\tilde{r}_{1}}(B_{n})}^{\tilde{r}_{1}} +\|a_{2} \|_{L^{\tilde{r}_{2}}(B_{n})}^{\tilde{r}_{2}} \bigr), \end{aligned}$$

where C is a positive constant independent of n. It turns out that

$$\begin{aligned} \frac{1}{2}\|v_{n}\|_{p,n}^{p}+\mu \|v_{n}\|_{q,n}^{q} \le&C \bigl( \|a_{0}\|_{L^{p^{\prime}}(B_{n})}^{p^{\prime}} +\|a_{1} \|_{L^{\tilde{r}_{1}}(B_{n})}^{\tilde{r}_{1}} +\|a_{2}\|_{L^{\tilde{r}_{2}}(B_{n})}^{\tilde{r}_{2}} \bigr) \\ \le& C \bigl(\|a_{0}\|_{L^{p^{\prime}}(\mathbb{R}^{N})}^{p^{\prime}} + \|a_{1}\|_{L^{\tilde{r}_{1}}(\mathbb{R}^{N})}^{\tilde{r}_{1}} +\|a_{2} \|_{L^{\tilde{r}_{2}}(\mathbb{R}^{N})}^{\tilde{r}_{2}} \bigr), \end{aligned}$$

whence (16) and (17) follow.

Fix \(m\in\mathbb{N}\). If \(n\geq m+1\), then by (16) we have

$$ \|v_{n}\|_{p,m+1}\leq\|v_{n} \|_{p,n}\le C. $$
(18)

Therefore, there exists \(v\in W^{1,p}(B_{m+1})\) such that

$$\begin{aligned}& v_{n} \rightharpoonup v \quad \mbox{in } W ^{1,p}(B_{m+1}), W^{1,q}(B_{m+1}), \end{aligned}$$
(19)
$$\begin{aligned}& v_{n}\to v \quad\mbox{in } L^{p}(B_{m+1}), L^{q}(B_{m+1}), \end{aligned}$$
(20)
$$\begin{aligned}& v_{n}(x)\rightarrow v(x) \quad \mbox{for a.e. } x\in B_{m+1} \end{aligned}$$
(21)

as \(n\to\infty\).

Let us show that \(v_{n}\) converges to v strongly in \(W ^{1,p}(B_{m})\) and \(W^{1,q}(B_{m})\). To this end, fix \(l\in\mathbb{N}\) and choose a smooth function \(\psi_{l}\) satisfying \(0\le\psi_{l} \le1\), \(\psi_{l}(r)=1\) if \(r\le m\), and \(\psi_{l}(r)=0\) if \(r\ge m+1/l\). Setting \(\eta_{l}(x):=\psi_{l}(|x|)\), we note that \((v_{n}-v)\eta_{l}\in W_{0}^{1,p}(B_{m+1})\subset W_{0}^{1,p}(B_{n})\) for any \(n\ge m+1\). Denote

$$V_{n}= \int_{B_{m}} \bigl(|\nabla v_{n}|^{p-2}+\mu| \nabla v_{n}|^{q-2} \bigr)\nabla v_{n} (\nabla v_{n}-\nabla v)\,dx. $$

Using \((v_{n}-v)\eta_{l}\) as a test function in \((\mathrm{P}_{n})\) and invoking the growth condition (F1), we obtain

$$\begin{aligned} V_{n} =& \int_{|x|< m+1/l} \bigl(f(x,v_{n},\nabla v_{n})- \lambda_{1}v_{n}^{p-1} -\mu\lambda_{2} v_{n}^{q-1} \bigr) (v_{n}-v)\eta_{l} \,dx \\ &{}- \int_{m\le|x|< m+1/l} \bigl(|\nabla v_{n}|^{p-2}+\mu| \nabla v_{n}|^{q-2}\bigr)\nabla v_{n} \nabla(v_{n}- v))\eta_{l}\,dx \\ &{}- \int_{m\le|x|< m+1/l} \bigl(|\nabla v_{n}|^{p-2}+\mu| \nabla v_{n}|^{q-2}\bigr)\nabla v_{n} \nabla \eta_{l} (v_{n}-v)\,dx \\ \le& \int_{B_{m+1}} \bigl(a_{0}(x)+a_{1}(x)v_{n}^{r_{1}}+a_{2}(x)| \nabla v_{n}|^{r_{2}} +\lambda_{1} v_{n}^{p-1}+\mu\lambda_{2} v_{n}^{q-1} \bigr) |v_{n}-v|\,dx \\ &{} + \int_{m\le|x|< m+1/l} \bigl(|\nabla v_{n}|^{p-1}+\mu| \nabla v_{n}|^{q-1} \bigr)|\nabla v|\,dx \\ &{}+d_{l} \int_{m\le|x|< m+1/l} \bigl(|\nabla v_{n}|^{p-1}+\mu| \nabla v_{n}|^{q-1} \bigr)|v_{n}-v|\,dx \\ \equiv& I_{n}^{1} +I_{n}^{2} +I_{n}^{3}, \end{aligned}$$

where \(d_{l}:=\sup_{|x|< m+1/l}|\nabla\eta_{l}(x)|\).

By Hölder’s inequality, (16) and (17), we have

$$\begin{aligned} I_{n}^{1} \le& \|v_{n}-v\|_{L^{p}(B_{m+1})} \bigl\{ \|a_{0}\|_{L^{p^{\prime}}(B_{m+1})} +\|a_{1}\|_{L^{\tilde{r}_{1}}(B_{m+1})} \|v_{n}\|_{L^{p}(B_{m+1})}^{r_{1}} \\ &{} +\|a_{2}\|_{L^{\tilde{r}_{2}}(B_{m+1})}\|\nabla v_{n} \|_{L^{p}(B_{m+1})}^{r_{2}} +\lambda_{1}\|v_{n} \|_{L^{p}(B_{m+1})}^{p-1} +\mu\lambda_{2} \|v_{n} \|_{L^{p^{\prime}(q-1)}(B_{m+1})}^{q-1} \bigr\} \\ \le& C_{1}\|v_{n}-v\|_{L^{p}(B_{m+1})}, \end{aligned}$$

where \(C_{1}\) is a positive constant independent of \(v_{n}\), n, m, and l. Again by Hölder’s inequality the following estimates follow:

$$\begin{aligned}& \begin{aligned}[b] I_{n}^{2} \le{}& \|\nabla v_{n}\|_{L^{p}(B_{m+1})}^{p-1} \biggl( \int_{m\le |x|< m+1/l} |\nabla v|^{p} \,dx \biggr)^{1/p} \\ &{} +\mu\|\nabla v_{n}\|_{L^{q}(B_{m+1})}^{q-1} \biggl( \int_{m\le |x|< m+1/l} |\nabla v|^{q} \,dx \biggr)^{1/q}, \end{aligned} \\& I_{n}^{3} \le d_{l}\|v_{n}-v \|_{L^{p}(B_{m+1})}\|\nabla v_{n}\|_{L^{p}(B_{m+1})}^{p-1} +d_{l}\mu\|v_{n}-v\|_{L^{q}(B_{m+1})}\|\nabla v_{n}\|_{L^{q}(B_{m+1})}^{q-1}. \end{aligned}$$

Thereby, from (16), (17), and (20) we derive

$$\begin{aligned} &\limsup_{n\to\infty} V_{n} \\ &\quad\le C^{p-1} \biggl( \int_{m\le|x|< m+1/l} |\nabla v|^{p} \,dx \biggr)^{1/p} +\mu C^{q-1} \biggl( \int_{m\le|x|< m+1/l} |\nabla v|^{q} \,dx \biggr)^{1/q} \end{aligned}$$

for all \(l\in\mathbb{N}\). Thus, letting \(l\to\infty\), we obtain that \(\limsup_{n\to\infty} V_{n}\le0\). As known from (19), \(v_{n}\) weakly converges to v in \(W^{1,p}(B_{m})\) and \(W^{1,q}(B_{m})\), so we may write

$$\begin{aligned} V_{n}+o(1) =& \int_{B_{m}} \bigl(|\nabla v_{n}|^{p-2}\nabla v_{n}-|\nabla v|^{p-2}\nabla v \bigr) (\nabla v_{n}-\nabla v)\,dx \\ &{} +\mu \int_{B_{m}} \bigl(|\nabla v_{n}|^{q-2}\nabla v_{n}-|\nabla v|^{q-2}\nabla v \bigr) (\nabla v_{n}-\nabla v)\,dx \\ \ge& \bigl(\|\nabla v_{n}\|_{L^{p}(B_{m})}^{p-1}-\|\nabla v\| _{L^{p}(B_{m})}^{p-1} \bigr) \bigl(\|\nabla v_{n} \|_{L^{p}(B_{m})}-\|\nabla v\|_{L^{p}(B_{m})} \bigr) \\ &{} +\mu \bigl(\|\nabla v_{n}\|_{L^{q}(B_{m})}^{p-1}-\| \nabla v\|_{L^{q}(B_{m})}^{q-1} \bigr) \bigl(\|\nabla v_{n} \|_{L^{q}(B_{m})}-\|\nabla v\|_{L^{q}(B_{m})} \bigr) \\ \ge& 0. \end{aligned}$$
(22)

What we have shown entails \(\lim_{n\to\infty} V_{n}=0\), \(\lim_{n\to \infty}\|\nabla v_{n}\|_{L^{p}(B_{m})}=\|\nabla v\|_{L^{p}(B_{m})}\) and \(\lim_{n\to\infty}\|\nabla v_{n}\|_{L^{q}(B_{m})}=\|\nabla v\|_{L^{q}(B_{m})}\) if \(\mu>0\). This implies that \(v_{n}\) converges to v strongly in \(W^{1,p}(B_{m})\) and \(W^{1,q}(B_{m})\) because the spaces \(W^{1,p}(B_{m})\) and \(W^{1,q}(B_{m})\) are uniformly convex.

Recalling that \(v_{n}>0\) in \(B_{m}\), we infer that v is a nonnegative solution of the problem

$$-\varDelta _{p} v+\lambda_{1} |v|^{p-2}v-\mu \varDelta _{q}v+\mu \lambda_{2} |v|^{q-2}v=f(x,v,\nabla v) \quad\mbox{in } B_{m}, v\ge 0 \mbox{ on } \partial B_{m}. $$

Now, by a diagonal argument and (21) there exist a relabeled subsequence of \(\{v_{n}\}\) and a function \(v\in W^{1,p}(\mathbb{R}^{N})\) such that

$$\begin{aligned}& v_{n} \to v \quad \mbox{in } W^{1,p}_{\mathrm{loc}}\bigl( \mathbb{R}^{N}\bigr), \\& v_{n}(x)\rightarrow v(x) \quad\mbox{for a.e } x\in \mathbb{R}^{N}. \end{aligned}$$

These convergence properties ensure that v is a solution of problem (P).

The next step in the proof is to show that v does not vanish in Ω. To do this, we fix \(m\in\mathbb{N}\) and a positive constant \(b_{m}\) satisfying \(b_{m}\le\inf_{x\in B_{m}} b_{0}(x)\), where the function \(b_{0}\) appears in assumption (F2). Moreover, choosing \(b_{m}\) even smaller, Lemma 3 provides a solution \(u_{m}\) of the problem

$$ \left \{ \textstyle\begin{array}{@{}l@{\quad}ll} -\varDelta _{p} u+\lambda_{1} |u|^{p-2}u-\mu \varDelta _{q}u+\mu\lambda_{2} |u|^{q-2}u =b_{m} u^{r_{0}} &\mbox{in } B_{m},\\ u>0&\mbox{in } B_{m},\\ u(x)= 0 &\mbox{on }\partial B_{m} \end{array}\displaystyle \right . $$

such that \(\|u_{m}\|_{L^{\infty}(B_{m})}\le\delta_{0}\), where \(\delta_{0}\) is given in assumption (F2). It follows from hypothesis (F2) that if \(t\le\|u_{m}\|_{L^{\infty}(B_{m})}\), then \(f(x,t,\xi)\ge b_{0}(x)t^{r_{0}}\) for all \(x\in\mathbb{R}^{N}\), \(\xi\in\mathbb{R}^{N}\). We are thus in a position to apply Theorem 4 to the functions \(u_{m}\) and \(v_{n}\) with \(n>m\) in place of \(u_{1}=u_{m}\) and \(u_{2}=v_{n}\), respectively, which renders \(v_{n}\ge u_{m}\) in \(B_{m}\) for every \(n>m\). This enables us to deduce that \(v\geq u_{m}\) in \(B_{m}\), so \(v(x)>0\) for almost every \(x\in\mathbb{R}^{N}\) because m was arbitrary.

Furthermore, since \(\lambda_{1}>0\) and \(v_{n}\) weakly converges to v in \(W^{1,p}(\mathbb{R}^{N})\) (we can extend \(v_{n}(x)=0\) if \(|x|\ge n\) (so \(\|v_{n}\|_{p,n}=\|\nabla v_{n}\|_{L^{p}(\mathbb{R}^{N})} +\lambda_{1} \|v_{n}\|_{L^{p}(\mathbb{R}^{N})}\))), by means of (16) and (17), we can check that \(v\in W^{1,p}(\mathbb{R}^{N})\cap W^{1,q}(\mathbb{R}^{N})\) if \(\mu>0\) and \(v\in W^{1,p}(\mathbb{R}^{N})\) if \(\mu=0\). According to the iteration process, it is proved that v is bounded on any bounded sets (see Section 5.1). Hence, the regularity theory as in [7] leads to \(v\in C^{1}_{\mathrm{loc}}(\mathbb{R}^{N})\). The proof of Theorem 1 is complete. □

5 Proof of Theorem 2

Throughout this section, we fix any (positive) solution v of (P) (belonging to \(W^{1,p}(\mathbb{R}^{N})\)). Define \(v_{M}:=\max\{v,M\}\) for \(M>0\). Here, we choose \(\overline{p}^{*}\) satisfying \(p^{2}<\overline{p}^{*}\) if \(N\le p\) and set \(\overline{p}^{*}=p^{*}=Np/(N-p)\) if \(N>p\). For \(R^{\prime}>R>0\), we take a smooth function \(\eta_{R,R^{\prime}}\) such that \(0\le\eta_{R,R^{\prime}}\le1\), \(\|\eta_{R,R^{\prime}}^{\prime}\|_{\infty}\le2/(R^{\prime}-R)\), \(\eta_{R,R^{\prime}}(t)=1\) if \(t\le R\), and \(\eta_{R,R^{\prime}}=0\) if \(t\ge R^{\prime}\).

5.1 Boundedness of solutions

Lemma 4

Let \(x_{0}\in\mathbb{R}^{N}\), \(M>0\), \(R^{\prime}>R>0\), \(\tilde{p}>1\), \(\gamma_{i}>1\), and \(1/\gamma_{i}+1/\gamma_{i}^{\prime}=1\) \((i=0, 1)\). Denote \(\eta(x):=\eta_{R,R^{\prime}}(|x-x_{0}|)\). Assume that \(\gamma_{i}^{\prime}\le\tilde{p}\) \((i=0,1)\) and \(v\in L^{\tilde{p}(p+\alpha)}(B(x_{0},R^{\prime}))\) with \(\alpha\ge0\). Then:

$$\begin{aligned}& \int_{B(x_{0},R^{\prime})} a_{0}vv_{M}^{\alpha} \eta^{p}\,dx \le\|a_{0}\|_{L^{\gamma_{0}}(B(x_{0},R^{\prime}))} \|v \|_{L^{\tilde{p}(p+\alpha)}(B(x_{0},R^{\prime}))}^{1+\alpha} B_{R^{\prime}}, \end{aligned}$$
(23)
$$\begin{aligned}& \int_{B(x_{0},R^{\prime})} a_{1} v^{r_{1}+1}v_{M}^{\alpha} \eta^{p}\,dx \le\|a_{1}\|_{L^{\gamma_{1}}(B(x_{0},R^{\prime}))} \|v \|_{L^{\tilde{p}(p+\alpha)}(B(x_{0},R^{\prime}))}^{r_{1}+1+\alpha} B_{R^{\prime}}, \end{aligned}$$
(24)

where \(B_{R^{\prime}} :=(1+|B(0,R^{\prime})|)\), and \(|B(0,R^{\prime})|\) denotes the Lebesgue measure of the ball \(B(0,R^{\prime})\). Moreover, if \(\gamma_{2}>p/(p-r_{2})\) and \(\gamma_{3}:=\frac{(p-r_{2})\gamma_{2}}{(p-r_{2})\gamma_{2}-p}\le\tilde{p}\), then

$$\begin{aligned} \int_{B(x_{0},R^{\prime})} a_{2} |\nabla v|^{r_{2}}vv_{M}^{\alpha} \eta^{p}\,dx \le&\frac{1}{4} \int_{B(x_{0},R^{\prime})} |\nabla v|^{p}v_{M}^{\alpha} \eta^{p}\,dx \\ &{}+ 4^{\frac{r_{2}}{p-r_{2}}}\|a_{2}\|_{L^{\gamma_{2}}(B(x_{0},R^{\prime}))}^{\frac {p}{p-r_{2}}} \|v \|_{L^{\tilde{p}(p+\alpha)}(B(x_{0},R^{\prime}))}^{\frac {p}{p-r_{2}}+\alpha} B_{R^{\prime}}. \end{aligned}$$
(25)

Proof

According to Hölder’s inequality, we easily show our assertions (23) and (24). So, we prove (25) only. By Young’s inequality and recalling that \(r_{2}< p-1\) and \(\eta^{p}\ge\eta ^{p^{2}/r_{2}}\), we have

$$\begin{aligned} \int_{B(x_{0},R^{\prime})} a_{2} |\nabla v|^{r_{2}}vv_{M}^{\alpha} \eta^{p}\,dx \le& \frac{1}{4} \int_{B(x_{0},R^{\prime})} |\nabla v|^{p}v_{M}^{\alpha} \eta^{p}\,dx \\ &{}+4^{\frac{r_{2}}{p-r_{2}}} \int_{B(x_{0},R^{\prime})} a_{2}^{\frac{p}{p-r_{2}}}v^{\frac{p}{p-r_{2}}} v_{M}^{\alpha}\,dx. \end{aligned}$$

Moreover, because of \(\gamma_{2}>p/(p-r_{2})\), \(p>p/(p-r_{2})\), and \(\tilde{p}\ge\gamma_{3}\), applying Hölder’s inequality, we obtain

$$\begin{aligned} \int_{B(x_{0},R^{\prime})} a_{2}^{\frac{p}{p-r_{2}}}v^{\frac{p}{p-r_{2}}} v_{M}^{\alpha}\,dx \le& \|a_{2}\|_{L^{\gamma_{2}}(B(x_{0},R^{\prime}))}^{\frac{p}{p-r_{2}}} \|v\|_{L^{\gamma_{3}(\frac{p}{p-r_{2}}+\alpha)}(B(x_{0},R^{\prime}))} ^{\frac{p}{p-r_{2}}+\alpha} \\ \le& \|a_{2}\|_{L^{\gamma_{2}}(B(x_{0},R^{\prime}))}^{\frac{p}{p-r_{2}}} \|v\|_{L^{\tilde{p}(p+\alpha)}(B(x_{0},R^{\prime}))} ^{\frac{p}{p-r_{2}}+\alpha}\bigl(1+\bigl|B\bigl(0,R^{\prime}\bigr)\bigr|\bigr). \end{aligned}$$

Hence, (25) follows. □

Lemma 5

Let \(x_{0}\in\mathbb{R}^{N}\), \(R^{\prime}>R>0\), \(\tilde{p}>1\), \(\gamma_{i}>1\), and \(1/\gamma_{i}+1/\gamma_{i}^{\prime}=1\) \((i=0,1)\). Assume that \(\gamma_{i}^{\prime}\le\tilde{p}\) \((i=0,1)\), \(\gamma_{2}>p/(p-r_{2})\), and \(\gamma_{3}:=\frac{(p-r_{2})\gamma_{2}}{(p-r_{2})\gamma_{2}-p}\le\tilde{p}\). If \(v\in L^{\tilde{p}(p+\alpha)}(B(x_{0},R^{\prime}))\) with \(\alpha\ge 0\), then

$$\begin{aligned} &\|v\|_{L^{\frac{\overline{p}^{*}}{p}(p+\alpha )}(B(x_{0},R))}^{p+\alpha} \le2^{p}(p+\alpha)^{p}C_{*}^{p}B_{R^{\prime}}(C_{R^{\prime}}+D_{R,R^{\prime}}) \max\bigl\{ 1,\|v\|_{L^{\tilde{p}(p+\alpha)}(B(x_{0},R^{\prime}))}\bigr\} ^{p+\alpha} \end{aligned}$$
(26)

with

$$\begin{aligned}& B_{R^{\prime}} :=\bigl(1+\bigl|B\bigl(0,R^{\prime}\bigr)\bigr|\bigr), \\& C_{R^{\prime}} := \bigl\{ \|a_{0}\|_{L^{\gamma_{0}}(B(x_{0},R^{\prime}))}+ \|a_{1}\|_{L^{\gamma _{1}}(B(x_{0},R^{\prime}))} +4^{\frac{r_{2}}{p-r_{2}}}\|a_{2} \|_{L^{\gamma_{2}}(B(x_{0},R^{\prime}))}^{\frac {p}{p-r_{2}}} \bigr\} , \\& D_{R,R^{\prime}} := \biggl\{ \frac{2^{3p-2}p^{p}+2^{p-1}}{(R^{\prime}-R)^{p}} +\frac{\mu2^{3q-2}p^{q}}{(R^{\prime}-R)^{q}} \biggr\} , \end{aligned}$$

where \(C_{*}\) is the positive constant from embedding from \(W^{1,p}(\mathbb{R}^{N})\) to \(L^{\overline{p}^{*}}(\mathbb{R}^{N})\).

Proof

Taking \(vv_{M}^{\alpha}\eta^{p}\in W^{1,p}_{0}(B(x_{0},R^{\prime}))\) (for \(M>0\)) as a test function, where \(\eta(x)=\eta_{R,R^{\prime}}(|x-x_{0}|)\), by Lemma 4 and (F1) we obtain

$$\begin{aligned} &\|a_{0}\|_{L^{\gamma_{0}}(B(x_{0},R^{\prime}))} \|v\|_{L^{\tilde{p}(p+\alpha)}(B(x_{0},R^{\prime}))}^{1+\alpha }B_{R^{\prime}} \\ &\qquad{}+\|a_{1}\|_{L^{\gamma_{1}}(B(x_{0},R^{\prime}))} \|v\|_{L^{\tilde{p}(p+\alpha)}(B(x_{0},R^{\prime}))}^{r_{1}+1+\alpha }B_{R^{\prime}} \\ &\qquad{} +4^{\frac{r_{2}}{p-r_{2}}}\|a_{2}\|_{L^{\gamma_{2}}(B(x_{0},R^{\prime}))}\|v\| _{L^{\tilde{p}(p+\alpha)}(B(x_{0},R^{\prime}))}^{\frac{p}{p-r_{2}}+\alpha} B_{R^{\prime}} +\frac{1}{4} \int_{B(x_{0},R^{\prime})} |\nabla v|^{p}v_{M}^{\alpha}\eta^{p}\,dx \\ &\quad\ge \int_{B(x_{0},R^{\prime})} |\nabla v|^{p}v_{M}^{\alpha}\eta^{p}\,dx +\lambda_{1} \int_{B(x_{0},R^{\prime})}v_{M}^{p+\alpha}\eta^{p} \,dx -\frac{2p}{R^{\prime}-R} \int_{B(x_{0},R^{\prime})} |\nabla v|^{p-1} v_{M}^{\alpha}v\eta^{p-1}\,dx \\ &\qquad{}+\mu \biggl\{ \int_{B(x_{0},R^{\prime})} |\nabla v|^{q}v_{M}^{\alpha}\eta^{p}\,dx -\frac{2p}{R^{\prime}-R} \int_{B(x_{0},R^{\prime})} |\nabla v|^{q-1} v_{M}^{\alpha}v\eta^{p-1}\,dx \biggr\} , \end{aligned}$$
(27)

where we use \(|\nabla\eta|\le2/(R^{\prime}-R)\). According to Young’s and Hölder’s inequalities, for \(j=p,q\), we see that

$$\begin{aligned} &\frac{2p}{R^{\prime}-R} \int_{B(x_{0},R^{\prime})} |\nabla v|^{j-1} v_{M}^{\alpha}v\eta^{p-1}\,dx \\ &\quad\le \frac{1}{4} \int_{B(x_{0},R^{\prime})} |\nabla v|^{j} v_{M}^{\alpha}\eta^{p}\,dx +\frac{2^{j}p^{j}4^{j-1}}{(R^{\prime}-R)^{j}} \int_{B(x_{0},R^{\prime})}v^{j+\alpha}\eta^{p-j}\,dx \\ &\quad\le \frac{1}{4} \int_{B(x_{0},R^{\prime})} |\nabla v|^{j} v_{M}^{\alpha}\eta^{p}\,dx +\frac{2^{3j-2}p^{j}}{(R^{\prime}-R)^{j}} \|v\|_{L^{\tilde{p}(p+\alpha)}(B(x_{0},R^{\prime}))}^{j+\alpha} B_{R^{\prime}}. \end{aligned}$$
(28)

Consequently, because of \(\mu\ge0\) and \(p+\alpha>r_{1}+1+\alpha,p/(p-r_{2})+\alpha\), it follows from (27) and (28) that

$$\begin{aligned} &B_{R^{\prime}} \biggl(C_{R^{\prime}} +\frac{2^{3p-2}p^{p}}{(R^{\prime}-R)^{p}} + \frac{\mu2^{3q-2}p^{q}}{(R^{\prime}-R)^{q}} \biggr) \max\bigl\{ 1,\|v\|_{L^{\tilde{p}(p+\alpha)}(B(x_{0},R^{\prime}))}\bigr\} ^{p+\alpha} \\ &\quad\ge \frac{1}{2} \int_{B(x_{0},R^{\prime})} |\nabla v|^{p}v_{M}^{\alpha}\eta^{p}\,dx +\lambda_{1} \int_{B(x_{0},R^{\prime})}v_{M}^{p+\alpha}\eta^{p} \,dx. \end{aligned}$$
(29)

Moreover, by using

$$\begin{aligned} &\bigl\| \nabla\bigl(v_{M}^{1+\alpha/p}\eta\bigr)\bigr\| _{L^{p}(\mathbb{R}^{N})}^{p}\\ &\quad\le2^{p-1} \bigl\{ \bigl\| \eta\nabla\bigl(v_{M}^{1+\alpha/p} \bigr)\bigr\| _{L^{p}(\mathbb{R}^{N})}^{p} +\bigl\| v_{M}^{1+\alpha/p}\nabla \eta\bigr\| _{L^{p}(\mathbb{R}^{N})}^{p} \bigr\} \\ &\quad\le 2^{p-1} \biggl(1+\frac{\alpha}{p} \biggr)^{p} \int_{B(x_{0},R^{\prime})} |\nabla v|^{p}v_{M}^{\alpha}\eta^{p}\,dx +\frac{2^{2p-1}}{(R^{\prime}-R)^{p}} \int_{B(x_{0},R^{\prime})}v_{M}^{p+\alpha}\,dx \end{aligned}$$

and Hölder’s inequality, due to the embedding from \(W^{1,p}(\mathbb{R}^{N})\) to \(L^{\overline{p}^{*}}(\mathbb{R}^{N})\), we have

$$\begin{aligned} &\frac{1}{2} \int_{B(x_{0},R^{\prime})} |\nabla v|^{p}v_{M}^{\alpha}\eta^{p}\,dx +\lambda_{1} \int_{B(x_{0},R^{\prime})}v_{M}^{p+\alpha}\eta^{p} \,dx \\ &\quad\ge2^{-p}p^{p}(p+\alpha)^{-p} \bigl\{ \bigl\| \nabla\bigl(v_{M}^{1+\alpha/p}\eta\bigr)\bigr\| _{L^{p}(\mathbb{R}^{N})}^{p} +\lambda_{1}\bigl\| v_{M}^{1+\alpha/p}\eta\bigr\| _{L^{p}(\mathbb{R}^{N})}^{p} \bigr\} \\ &\qquad{} -\frac{2^{p-1}p^{p}}{(p+\alpha)^{p}(R^{\prime}-R)^{p}} \int_{B(x_{0},R^{\prime})}v_{M}^{p+\alpha}\,dx \\ &\quad\ge2^{-p}p^{p}(p+\alpha)^{-p} \bigl\| v_{M}^{1+\alpha/p}\eta\bigr\| _{W^{1,p}(\mathbb{R}^{N})}^{p} \\ &\qquad{} -\frac{2^{p-1}}{(R^{\prime}-R)^{p}} \|v\|_{L^{\tilde{p}(p+\alpha)}(B(x_{0},R^{\prime}))}^{p+\alpha }\bigl(1+\bigl|B \bigl(0,R^{\prime}\bigr)\bigr|\bigr) \\ &\quad\ge 2^{-p}p^{p}(p+\alpha)^{-p}C_{*}^{-p} \bigl\| v_{M}^{1+\alpha/p}\eta\bigr\| _{L^{\overline{p}^{*}}(\mathbb{R}^{N})}^{p} - \frac{2^{p-1}}{(R^{\prime}-R)^{p}} \|v\|_{L^{\tilde{p}(p+\alpha)}(B(x_{0},R^{\prime}))}^{p+\alpha }B_{R^{\prime}} \\ &\quad\ge2^{-p}p^{p}(p+\alpha)^{-p}C_{*}^{-p} \|v_{M}\|_{L^{\overline{p}^{*}(p+\alpha)/p}(B(x_{0},R))}^{p+\alpha} \\ &\qquad{} -\frac{2^{p-1}}{(R^{\prime}-R)^{p}} \|v\|_{L^{\tilde{p}(p+\alpha)}(B(x_{0},R^{\prime}))}^{p+\alpha} B_{R^{\prime}}. \end{aligned}$$
(30)

Therefore, (29) and (30) lead to

$$\begin{aligned} &2^{-p}p^{p}(p+\alpha)^{-p}C_{*}^{-p} \|v_{M}\|_{L^{\overline{p}^{*}(p+\alpha)/p}(B(x_{0},R))}^{p+\alpha} \\ &\quad\le B_{R^{\prime}}(C_{R^{\prime}}+D_{R,R^{\prime}}) \max\bigl\{ 1,\|v \|_{L^{\tilde{p}(p+\alpha)}(B(x_{0},R^{\prime}))}\bigr\} ^{p+\alpha}. \end{aligned}$$
(31)

Applying Fatou’s lemma and letting \(M\to\infty\) in (31), our conclusion follows. □

Proposition 1

Under the assumptions in Theorem 2, we have that \(v\in L^{\infty}(\mathbb{R}^{N})\).

Proof

First, in the case of \(N>p\), we note that

$$\begin{aligned}& \gamma_{j}^{\prime}< \frac{p^{*}}{p} \quad\Longleftrightarrow \quad \gamma_{j}>\frac{p^{*}}{p^{*}-p}\quad (j=0,1), \\& \gamma_{2}>\frac{p}{p-r_{2}}\quad \mbox{and} \quad \gamma_{3}:=\frac{(p-r_{2})\gamma_{2}}{(p-r_{2})\gamma_{2}-p}< \frac{p^{*}}{p} \quad \Longleftrightarrow\quad \gamma_{2}>\frac{pp^{*}}{(p-r_{2})(p^{*}-p)}. \end{aligned}$$

In the cases of (i) and (ii) (case \(p<\overline{p}^{*}/p\)), we take \(\gamma_{0}=p^{\prime}\) and \(\gamma_{j}=\tilde{r}_{j}\) (\(j=1,2\)). Then, we have \(\gamma_{0}^{\prime}=p\), \(\gamma_{1}^{\prime}=\tilde{r}_{1}^{\prime}=p/(r_{1}+1)\le p\), \(\gamma_{2}=\tilde{r}_{2}=p/(p-r_{2}-1)>p/(p-r_{2})\), and \(\gamma_{3}=(p-r_{2})\tilde{r_{2}}/((p-r_{2})\tilde{r}_{2}-p)=p-r_{2}\le p\). Choose such that

$$\begin{aligned}& \bigl(\max\bigl\{ \gamma_{0}^{\prime},\gamma_{1}^{\prime}, \gamma_{3}\bigr\} = \bigr) \tilde{p}=p \biggl(< \frac{\overline{p}^{*}}{p}\biggr) \mbox{ in the cases of (i) and (ii)}, \\& \max\bigl\{ \gamma_{0}^{\prime},\gamma_{1}^{\prime}, \gamma_{3}\bigr\} \le \tilde{p}< \frac{p^{*}}{p} \mbox{ in the case of (iii)}. \end{aligned}$$

Let \(R_{*}\) be the positive constant satisfying (4) in the case of (iii) and any positive constant in the cases of (i) and (ii). Put

$$ A_{i}:=\left \{ \textstyle\begin{array}{@{}l@{\quad}l} \|a_{i}\|_{L^{\gamma_{i}}(\mathbb{R}^{N})} & \mbox{if (i) and (ii)}, \\ \sup_{x\in\mathbb{R}^{N}} \|a_{i}\|_{L^{\gamma_{i}}(B(x,2R_{*}))} &\mbox{if (iii)} \end{array}\displaystyle \right . $$

for \(i=0,1,2\). Define the sequences \(\{\alpha_{n}\}\), \(\{R_{n}^{\prime}\}\), and \(\{R_{n}\}\) by

$$\begin{aligned}& \alpha_{0}:=\frac{\overline{p}^{*}}{\tilde{p}}-p>0,\qquad \tilde{p}(p+ \alpha_{n+1})=\frac{\overline{p}^{*}}{p} (p+\alpha_{n}), \\& R_{n}^{\prime}:=\bigl(1+2^{-n}\bigr)R_{*}, \qquad R_{n}:=R_{n+1}^{\prime}. \end{aligned}$$

Recall that \(v\in W^{1,p}(\mathbb{R}^{N})\), and using the embedding of \(W^{1,p}(\mathbb{R}^{N})\) to \(L^{\overline{p}^{*}}(\mathbb{R}^{N})\), we see that \(v\in L^{\overline{p}^{*}}(\mathbb{R}^{N})=L^{\tilde{p}(p+\alpha _{0})}(\mathbb{R}^{N})\).

Fix any \(x_{0}\in\mathbb{R}^{N}\). Then Lemma 5 guarantees that if \(v\in L^{\tilde{p}(p+\alpha_{n})}(B(x_{0},R_{n}^{\prime}))\), then \(v\in L^{\frac{\overline{p}^{*}}{p}(p+\alpha_{n})}(B(x_{0},R_{n})) =L^{\tilde{p}(p+\alpha_{n+1})}(B(x_{0},R_{n+1}^{\prime}))\). Noting that

$$\begin{aligned}& B_{R^{\prime}_{n}} \le\bigl(1+\bigl|B(0,2R_{*})\bigr|\bigr)=:B_{0}, \\& C_{R_{n}^{\prime}} \le A_{0}+A_{1}+4^{\frac{r_{2}}{p-r_{2}}}A_{2}^{\frac {p}{p-r_{2}}}+1=:C_{0}, \\& D_{R_{n},R_{n}^{\prime}} \le\frac{(1+p^{p})2^{p(n+3)}}{R_{*}^{p}} +\frac{\mu q^{q}2^{q(n+3)}}{R_{*}^{q}}=:D_{n} \le C^{\prime}2^{p(n+3)} \end{aligned}$$

for any \(n\ge0\) with sufficiently large \(C^{\prime}\) independent of n and setting

$$b_{n}:=\max\bigl\{ 1,\|v\|_{L^{\tilde{p}(p+\alpha_{n})(B(x_{0},R_{n}^{\prime}))}}\bigr\} , $$

by Lemma 5 we obtain

$$ b_{n+1}\le C^{\frac{1}{p+\alpha_{n}}}(p+\alpha_{n})^{\frac{p}{p+\alpha_{n}}} (C_{0}+D_{n})^{\frac{1}{p+\alpha_{n}}}b_{n} $$
(32)

for every \(n\ge0\) with \(C:=2^{p}(C_{*}+1)^{p}B_{0}\). Put \(P:=\tilde{p}p/\overline{p}^{*}<1\). Then, because of \(p+\alpha_{n+1}=(p+\alpha_{n})/P\), \(\alpha_{n+1}>\alpha_{n}/P>\alpha_{0}(1/P)^{n+1}\to\infty\) as \(n\to \infty\). Moreover, we see that

$$\begin{aligned} &S_{1} :=\sum_{n=0}^{\infty}\frac{1}{p+\alpha_{n}}= \frac{1}{p+\alpha_{0}}\sum_{n=0}^{\infty}P^{n} =\frac{1}{(p+\alpha_{0})(1-P)}< \infty, \\ &S_{2} :=\ln \prod_{n=0}^{\infty}(p+ \alpha_{n})^{\frac{p}{p+\alpha_{n}}} =\frac{p}{p+\alpha_{0}}\sum _{n=0}^{\infty}P^{n} \bigl(\ln (p+ \alpha_{0})+n\ln P^{-1} \bigr)< \infty, \end{aligned}$$

and

$$\begin{aligned} S_{3}&:=\ln \prod_{n=0}^{\infty}(C_{0}+D_{n})^{\frac{1}{p+\alpha_{n}}} =\sum_{n=0}^{\infty}\frac{P^{n}}{p+\alpha_{0}} \ln (C_{0}+D_{n}) \\ &\le\sum_{n=0}^{\infty}\frac{P^{n}}{p+\alpha_{0}} p(n+3)\ln \bigl(C_{0}+C^{\prime}\bigr)2< \infty. \end{aligned}$$

As a result, by iteration in (32) and the equality \(\tilde{p}(p+\alpha_{0})=\overline{p}^{*}\) we obtain

$$\|v\|_{L^{\frac{\overline{p}^{*}}{p}(p+\alpha_{n})}(B(x_{0},R_{*}))} \le b_{n} \le C^{S_{1}}e^{S_{2}}e^{S_{3}} \max\bigl\{ 1,\|v\|_{L^{\overline{p}^{*}}(B(x_{0},2R_{*}))}\bigr\} $$

for every \(n\ge1\). Letting \(n\to\infty\), this ensures that

$$ \|v\|_{L^{\infty}(B(x_{0},R_{*}))} \le C^{S_{1}}e^{S_{2}}e^{S_{3}} \max\bigl\{ 1,\|v\|_{L^{\overline{p}^{*}}(B(x_{0},2R_{*}))}\bigr\} . $$
(33)

Recalling that \(v\in W^{1,p}(\mathbb{R}^{N})\) and using the embedding of \(W^{1,p}(\mathbb{R}^{N})\) to \(L^{\overline{p}^{*}}(\mathbb{R}^{N})\), (33) yields that

$$\begin{aligned} \|v\|_{L^{\infty}(B(x_{0},R_{*}))} &\le C^{S_{1}}e^{S_{2}}e^{S_{3}} \max\bigl\{ 1,\|v\|_{L^{\overline{p}^{*}}(\mathbb{R}^{N})}\bigr\} \\ &\le C^{S_{1}}e^{S_{2}}e^{S_{3}} \max\bigl\{ 1,C_{*}\|v \|_{W^{1,p}(\mathbb{R}^{N})}\bigr\} , \end{aligned}$$

whence v is bounded in \(\mathbb{R}^{N}\) because \(x_{0}\in\mathbb{R}^{N}\) is arbitrary and the constant \(C^{S_{1}}e^{pS_{2}}e^{S_{3}}\) is independent of \(x_{0}\). □

5.2 Proof of Theorem 2

Proof of Theorem 2

Since v is bounded in \(\mathbb{R}^{N}\) by Proposition 1, we put \(M_{0}:=\|v\|_{L^{\infty}(\mathbb{R}^{N})}\). Then, as in Lemma 4, we see that

$$\begin{aligned} & \int_{B(x_{0},R^{\prime})} a_{0}vv_{M}^{\alpha} \eta^{p}\,dx \le\|a_{0}\|_{L^{\gamma_{0}}(B(x_{0},R^{\prime}))} M_{0} \|v\|_{L^{\tilde{p}(p+\alpha)}(B(x_{0},R^{\prime}))}^{\alpha} B_{R^{\prime}}, \end{aligned}$$
(34)
$$\begin{aligned} & \int_{B(x_{0},R^{\prime})} a_{1} v^{r_{1}+1}v_{M}^{\alpha} \eta^{p}\,dx \le\|a_{1}\|_{L^{\gamma_{1}}(B(x_{0},R^{\prime}))} M_{0}^{1+r_{1}}\|v\|_{L^{\tilde{p}(p+\alpha)}(B(x_{0},R^{\prime}))}^{\alpha} B_{R^{\prime}}, \end{aligned}$$
(35)
$$\begin{aligned} &\begin{aligned}[b] & \int_{B(x_{0},R^{\prime})} a_{2} |\nabla v|^{r_{2}}vv_{M}^{\alpha} \eta^{p}\,dx \\ &\quad \le\frac{1}{4} \int_{B(x_{0},R^{\prime})} |\nabla v|^{p}v_{M}^{\alpha} \eta^{p}\,dx + 4^{\frac{r_{2}}{p-r_{2}}}\|a_{2}\|_{L^{\gamma_{2}}(B(x_{0},R^{\prime}))}^{\frac {p}{p-r_{2}}} M_{0}^{\frac{p}{p-r_{2}}}\|v\|_{L^{\tilde{p}(p+\alpha)}(B(x_{0},R^{\prime}))}^{\alpha} B_{R^{\prime}}, \end{aligned} \end{aligned}$$
(36)

and

$$ \|v\|_{L^{\tilde{p}(p+\alpha)}(B(x_{0},R^{\prime}))}^{j+\alpha}\le M_{0}^{j}\|v \|_{L^{\tilde{p}(p+\alpha)}(B(x_{0},R^{\prime}))}^{\alpha}\quad (j=p,q) . $$
(37)

Fix any \(x_{0}\in\mathbb{R}^{N}\). It follows from the argument as in the proof of Lemma 5 with (34), (35), (36), and (37) that

$$\begin{aligned} &\|v\|_{L^{\frac{\overline{p}^{*}}{p}(p+\alpha )}(B(x_{0},R))}^{p+\alpha} \le2^{p}(p+\alpha)^{p}C_{*}^{p}B_{R^{\prime}}(C_{R^{\prime}}+D_{R,R^{\prime}}) (M_{0}+1)^{p} \|v\|_{L^{\tilde{p}(p+\alpha)}(B(x_{0},R^{\prime}))}^{\alpha}, \end{aligned}$$
(38)

provided that \(v\in L^{\tilde{p}(p+\alpha)}(B(x_{0},R^{\prime}))\). Choose \(\gamma_{i}\) \((i=0,1,2)\) and and define the sequences \(\{\alpha_{n}\}\), \(\{R_{n}^{\prime}\}\), and \(\{R_{n}\}\) as in the proof of Proposition 1. Set

$$V_{n}:=\|v\|_{L^{\tilde{p}(p+\alpha_{n})}(B(x_{0},R^{\prime}_{n}))}^{\alpha_{n}}. $$

Then, by the same argument as in the proof of Proposition 1 with (38) we obtain

$$ V_{n}^{\frac{p+\alpha_{n-1}}{\alpha_{n}}} \le C (p+\alpha_{n-1})^{p}(C_{0}+D_{n-1}) V_{n-1} $$
(39)

with \(C:=2^{p}C_{*}^{p}B_{0}(M_{0}+1)^{p}\). Recall that

$$\alpha_{n}+p=P^{-1}(p+\alpha_{n-1}) \quad \mbox{and}\quad \frac{p}{p+\alpha_{0}}=P. $$

Define

$$Q_{n}:=\prod_{k=2}^{n+1} \biggl(1+\frac{P^{k}}{1-P^{k}} \biggr) = \prod_{k=2}^{n+1} \bigl(1-P^{k} \bigr)^{-1} \quad \mbox{and}\quad W_{n}:=(C_{0}+D_{n}). $$

Then, inequality (39) leads to

$$\begin{aligned} \ln V_{n} \le{}&\frac{\alpha_{n}}{p+\alpha_{n-1}} \bigl(\ln V_{n-1}+\ln C(p+\alpha_{n-1})^{p}+\ln W_{n-1} \bigr) \\ ={}&P^{-1} \bigl(1-P^{n+1} \bigr) \bigl(\ln V_{n-1}+ p\ln C P^{-n+1}(p+\alpha_{0})+\ln W_{n-1} \bigr) \\ \le{}& P^{-1} \bigl(1-P^{n+1} \bigr)\ln V_{n-1} +pP^{-1}\ln (C+1) P^{-n+1}(p+\alpha_{0})+P^{-1} \ln W_{n-1} \\ \le{}& P^{-n} \Biggl( \prod _{k=1}^{n} \bigl(1-P^{k+1} \bigr) \Biggr) \ln V_{0} +p\sum_{k=1}^{n} P^{-k}\ln (C+1) P^{-n+k}(p+\alpha_{0}) \\ &{} +\sum_{k=1}^{n} P^{-k}\ln W_{n-k} \\ ={}& P^{-n}Q_{n}^{-1} \ln V_{0} +p \sum_{k=1}^{n} P^{-k}\ln (C+1) P^{-n+k}(p+\alpha_{0}) +\sum_{k=1}^{n} P^{-k}\ln W_{n-k} \end{aligned}$$

for every n because of \(\ln (C+1) P^{-n+1}(p+\alpha_{0})> 0\) and \(\ln W_{n}>0\) for all n. Therefore, we have

$$\begin{aligned} &\ln \|v\|_{L^{\tilde{p}(p+\alpha_{n})}(B(x_{0},R^{\prime}_{n}))} \\ &\quad=\frac{\ln V_{n}}{\alpha_{n}}=\frac{P^{n}\ln V_{n}}{p+\alpha_{0}-pP^{n}} \\ &\quad\le\frac{Q_{n}^{-1}\ln V_{0}}{p+\alpha_{0}-pP^{n}} +\frac{\sum_{l=0}^{n-1} P^{l} \ln (C+1) P^{-l}(p+\alpha _{0})}{p+\alpha_{0}-pP^{n}} +\frac{\sum_{l=0}^{n-1} P^{l}\ln W_{l}}{p+\alpha_{0}-pP^{n}}. \end{aligned}$$
(40)

Here, taking a sufficiently large positive constant \(C^{\prime}\) independent of n, we see that

$$\sum_{l=0}^{n-1} P^{l} \ln (C+1) P^{-l}(p+\alpha_{0}) \le C^{\prime}\sum _{l=0}^{\infty}P^{l} (l+1)=:S_{1}< \infty $$

and

$$\sum_{l=0}^{n-1} P^{l}\ln W_{l}\le C^{\prime}\sum_{l=0}^{n-1} P^{l} (l+3)\le C^{\prime}\sum_{l=0}^{\infty}P^{l} (l+3) =:S_{2}< \infty. $$

Next, we shall show that \(\{Q_{n}\}\) is a convergent sequence. It is easy see that \(\{Q_{n}\}\) is increasing. Moreover, setting \(d_{k}:=\ln (1+\frac{P^{k}}{1-P^{k}} )\), we see that

$$\lim_{k\to\infty}\frac{d_{k+1}}{d_{k}} = \lim_{k\to\infty} \frac{\ln (1-P^{k+1})}{\ln (1-P^{k})} =\lim_{k\to\infty}\frac{1-P^{k}}{1-P^{k+1}} P=P< 1 $$

by L’Hospital’s rule. This implies that

$$\ln Q_{n}=\sum_{k=2}^{n+1} \ln \biggl(1+\frac {P^{k}}{1-P^{k}} \biggr) \le\sum_{k=1}^{\infty}\ln \biggl(1+\frac{P^{k}}{1-P^{k}} \biggr) < \infty. $$

Therefore, \(\{Q_{n}\}\) is bounded from above, whence \(\{Q_{n}\}\) converges, and

$$1< \frac{1}{1-P^{2}}=Q_{1}\le Q_{\infty}:=\lim _{n\to\infty}Q_{n}< \infty. $$

Consequently, letting \(n\to\infty\) in (40), we have

$$\|v\|_{L^{\infty}(B(x_{0},R_{*}))} \le(pS_{1}S_{2})^{\frac{1}{p+\alpha_{0}}} \|v \|_{L^{\overline{p}^{*}}(B(x_{0},2R_{*}))}^{\frac{\alpha_{0}}{(p+\alpha _{0})Q_{\infty}}}. $$

This yields our conclusion since \(\|v\|_{L^{\overline{p}^{*}}(B(x_{0},2R_{*}))}\to0\) as \(|x_{0}|\to \infty\), \(\alpha_{0}>0\), and the constant \(pS_{1}S_{2}\) is independent of \(x_{0}\). □